Đến nội dung

dark templar nội dung

Có 1000 mục bởi dark templar (Tìm giới hạn từ 06-06-2020)



Sắp theo                Sắp xếp  

#371476 Topic bất đẳng thức THCS (2)

Đã gửi bởi dark templar on 22-11-2012 - 12:09 trong Bất đẳng thức và cực trị

bai nua
cho a,b,c la 3 canh 1 tam giac. cmr
$a^{2}(b+c-a)+b^{2}(c+a-b)+c^{2}(a+b-c)\leq 3abc$

Gõ tiếng Việt có dấu,bạn nhé :)
Nhìn chung thì bài này có 2 hướng tiếp cận:
+Về mặt Hình học:
Nó xuất phát từ BĐT $\cos{A}+\cos{B}+\cos{C} \le \frac{3}{2}$ hay 1 dạng tương đương khác quen thuộc hơn là $R \ge 2r$.
+Về mặt Đại Số:
Ta có 1 dạng phát biểu khác "dễ nhìn" hơn cho BĐT này:
$$(a+b-c)(b+c-a)(c+a-b) \le abc$$
Và đây là hệ quả của AM-GM:
$$(a+b-c)(b+c-a) \le b^2$$
Thật ra phép biến đổi về Schur cũng không quá khó hiểu vì đó chính là bản chất bài toán này.

P/s:Mod THCS xóa giùm mấy bài post của Nguyen Tho The Cuong giùm,post trùng lặp nhiều quá :(



#394306 [Giải trí]Cặp đôi hoàn hảo VMF 2013

Đã gửi bởi dark templar on 07-02-2013 - 13:35 trong Góc giao lưu

sao thiếu nhiều điều kiện thế em.Thôi cứ vơ tạm con bạn thân nào đấy(Nó phải đồng ý đấy nhá) post lên đây mà dự thi :)

Không sao đâu thím Hoàng,cứ để em nó up ,nhưng BGK sẽ không bồi thường tiền thuốc men cho người post >:)

Okay,tks chị

Lại 1 bằng chứng nữa củng cố mối nghi ngờ: Hoàng là con gái :ohmy:

Còn snowwhite post tấm ảnh ở #225 không khai rõ tên tuổi thì sao update ? Mà các thí sinh cũng lưu ý là nếu chụp cùng với GF của mình thì sẽ được tuyển thẳng qua vòng 1,đã được thím Hoàng nêu rõ ở #1. ;)



#393951 [Giải trí]Cặp đôi hoàn hảo VMF 2013

Đã gửi bởi dark templar on 06-02-2013 - 19:25 trong Góc giao lưu

Anh Hân xuất quân luôn đi chứ ạ :D

BGK không xuất quân được :P Hân vào BGK cũng vì FA nặng :))



#394012 [Giải trí]Cặp đôi hoàn hảo VMF 2013

Đã gửi bởi dark templar on 06-02-2013 - 20:26 trong Góc giao lưu

Mình men 100% nha ko phải xoắn

Men với chả không,chú có gì để chứng minh thế :))



#395385 [Giải trí]Cặp đôi hoàn hảo VMF 2013

Đã gửi bởi dark templar on 10-02-2013 - 00:03 trong Góc giao lưu

Up đề toán của cuộc thi "Cặp đôi hoàn hảo 2013".Các cặp vào xem đề nhanh nhé :)
File gửi kèm  Talent.pdf   177.46K   230 Số lần tải

P.s:Lời văn có bựa quá cũng đừng chém BGk :P



#395520 [Giải trí]Cặp đôi hoàn hảo VMF 2013

Đã gửi bởi dark templar on 10-02-2013 - 17:47 trong Góc giao lưu

Èo! Em chưa học lô-ga-rít! Làm ăn kiểu gì bây giờ! :(

Thế chú học hàm mũ chưa,logarit là ngược lại với hàm mũ đó ;)



#395389 [Giải trí]Cặp đôi hoàn hảo VMF 2013

Đã gửi bởi dark templar on 10-02-2013 - 00:10 trong Góc giao lưu

Thôi e xin rút a ạ :P

Đã đi thi thì thi tới bến luôn đi em,chủ yếu là tinh thần vui mừng năm mới thôi :)



#393825 [Giải trí]Cặp đôi hoàn hảo VMF 2013

Đã gửi bởi dark templar on 06-02-2013 - 16:41 trong Góc giao lưu

#108 ảnh hơi tối nhỉ ? Căng mắt ra mới thấy được cái mặt :| Cu Đạt vớ đâu được " bạn" xinh thế :))
P.s:Mới vắng có buổi sáng thôi mà mấy bác chém kinh thế,tăng đến 4 trang :mellow:



#395271 [Giải trí]Cặp đôi hoàn hảo VMF 2013

Đã gửi bởi dark templar on 09-02-2013 - 17:26 trong Góc giao lưu

Hân và GF ko định dự thi à :)), anh vote cho ;)

Có trưa nào a thấy chu đi ăn một mình đâu nhỉ :))

Em cứ để Hân tiếp tục cái "Mission Impossible" của mình đi ,dự là đến năm 12 mới thành "chính thức" được :))

P.s:@Hân:Tết này có định đi chơi đâu với ai đó không :)



#393316 [Giải trí]Cặp đôi hoàn hảo VMF 2013

Đã gửi bởi dark templar on 05-02-2013 - 10:21 trong Góc giao lưu

Đẹp đôi quá.Tiếc là chỉ là bạn thân :luoi:

Đồng ý :namtay Chú Trọng có định tiến tới không ? Anh em VMF luôn ủnh hộ chú :))
P.s:Pu muốn tham gia cũng phải thep cặp nhé :closedeyes:



#393336 [Giải trí]Cặp đôi hoàn hảo VMF 2013

Đã gửi bởi dark templar on 05-02-2013 - 11:10 trong Góc giao lưu

À, em hỏi ý Pu thì nó nói muốn thi tiếp, thì liệu nó được bắt cặp với em hay với zai khác vậy mấy anh ?

Thế là em muốn một mình bao thầu 2 em àh :mellow: Phải bắt cặp với zai khác mới gọi là cạnh tranh chứ :))



#393438 [Giải trí]Cặp đôi hoàn hảo VMF 2013

Đã gửi bởi dark templar on 05-02-2013 - 18:33 trong Góc giao lưu

Diễn đàn mình có nhiều bàn nữ xinh mình biết mà :).Chúc mừng Việt có gấu hờ :luoi:

Phim giả tình thật ai biết được ;) Như Việt nói :"Biết đâu sẽ có bất ngờ ? " :))



#310407 Phương trình và hệ phương trình qua các đề thi thử Đại học 2012

Đã gửi bởi dark templar on 14-04-2012 - 21:27 trong Phương trình - hệ phương trình - bất phương trình

Bài 37: Giải hệ phương trình sau:
$$\left\{\begin{matrix} 2^{2y-x}+2^{y}=2^{x+1} \\ \log_5{(x^2+3y+1)}-\log_5{y}=-2x^2+4y-1 \end{matrix}\right.$$

Đề thi thử của MathVN.com



#626431 Inequalities From 2016 Mathematical Olympiads

Đã gửi bởi dark templar on 10-04-2016 - 20:00 trong Bất đẳng thức - Cực trị

Bất đẳng thức này đúng là quen thuộc, có thể tham khảo thêm đề Việt Nam MO 1998.

 

Bài 19 (Cyprus TST). Giả sử $a,\,b,\,c$ là độ dài ba cạnh của tam giác $ABC$ và thỏa mãn điều kiện
$$a\sqrt{8}+b\sqrt{6}+c\sqrt{2}\ge 4\sqrt{a^2+b^2+c^2},$$
Chứng minh rằng $ABC$ là tam giác vuông.

 

 

Theo C-S:

$$4\sqrt{a^{2}+b^{2}+c^{2}}=\sqrt{8+6+2}\sqrt{a^{2}+b^{2}+c^{2}}\geqslant a\sqrt{8}+b\sqrt{6}+c\sqrt{2}$$

 

Kết hợp với điều kiện đầu bài cho,ta sẽ có đẳng thức xảy ra hay:

$$\frac{a}{\sqrt{8}}=\frac{b}{\sqrt{6}}=\frac{c}{\sqrt{2}}\Leftrightarrow \frac{a^{2}}{8}=\frac{b^{2}}{6}=\frac{c^{2}}{2}=\frac{b^{2}+c^{2}}{8}\Rightarrow a^{2}=b^{2}+c^{2}$$

 

Vậy tam giác ABC vuông.

 

Bài 15 (Australien MO). Cho $a,b$ là hai số thực thỏa mãn $a^{2}+b^{2}=1.$ Chứng minh rằng

$$\left | a+\frac{a}{b}+b+\frac{b}{a} \right |\geq 2-\sqrt{2}.$$

 

Theo BĐT trị tuyệt đối và C-S:

$$\left | (a+b)+\left ( \frac{a}{b}+\frac{b}{a} \right ) \right |\geqslant \left | \frac{a}{b}+\frac{b}{a} \right |-\left | a+b \right |\geqslant 2-\sqrt{2(a^{2}+b^{2})}=2-\sqrt{2}$$




#625078 Inequalities From 2016 Mathematical Olympiads

Đã gửi bởi dark templar on 05-04-2016 - 17:20 trong Bất đẳng thức - Cực trị

 

Bài 12 (China Junior High School Mathematics League). Với $x,\,y,\,z$ là ba số thực dương thỏa mãn $xy+yz+zx\neq 1$ và $\frac{(x^2-1)(y^2-1)}{xy} +\frac{(y^2-1)(z^2-1)}{yz} +\frac{(z^2-1)(x^2-1)}{zx} =4.$ Prove that $$9(x+y)(y+z)(z+x)\geqslant 8xyz(xy+yz+zx).$$

Biến đổi điều kiện đề bài cho ,ta sẽ có:

$\sum x\left ( y^{2}-1 \right )\left ( z^{2}-1 \right )=4xyz\Leftrightarrow xyz\sum xy+\sum x-\sum xy(x+y)=4xyz$

$\Leftrightarrow xyz\sum xy+\left ( \sum x-xyz \right )=\left ( \sum x \right )\left ( \sum xy \right )\Leftrightarrow \left ( \sum xy -1 \right )\left ( \sum x -xyz\right )=0$

$\Leftrightarrow x+y+z=xyz$

 

Đổi biến $\left ( a,b,c \right )\rightarrow \left ( \frac{1}{x},\frac{1}{y},\frac{1}{z} \right )$ thì ta có $a,b,c>0$ và $ab+bc+ca=1$.

 

BĐT cần chứng minh trở thành:

$$9(a+b)(b+c)(c+a)\geqslant 8(a+b+c)$$

 

Viết dưới dạng thuần nhất :

$$\left ( a+b \right )(b+c)(c+a)\geqslant \frac{8}{9}(a+b+c)(ab+bc+ca)$$

 

BĐT này chỉ là hệ quả của $(a+b)(b+c)(c+a)=(a+b+c)(ab+bc+ca)-abc$ và $abc\leqslant \frac{(a+b+c)(ab+bc+ca)}{9}$.Ta có đpcm.




#624984 Inequalities From 2016 Mathematical Olympiads

Đã gửi bởi dark templar on 05-04-2016 - 08:26 trong Bất đẳng thức - Cực trị

 Bài 2. Ta chú ý đến một phân tích quen thuộc là $(x+y)^2=(x-y)^2+4xy$

 Bây giờ đặt $\sum _{j=1}^na_jb_j=S$ và $a_{i}b_{i}=S_i$ thì ta cần chứng minh $\sum_{i=1}^n \dfrac{(a_{i+1}+b_{i+1})^2}{n(a_i-b_i)^2+4(n-1)S} \geq \dfrac{1}{n-1}$

 Hay $\sum_{i=1}^n \dfrac{(a_{i+1}-b_{i+1})^2+4S_{i+1}}{n(a_i-b_i)^2+4(n-1)S} \geq \dfrac{1}{n-1}$

 Không mất tính tổng quát giả sử $(a_1-b_1)^2\leq \min \left \{(a_2-b_2)^2,(a_3-b_3)^2,...,(a_n-b_n)^2\right \}$ thì ta có

 $\sum_{i=1}^n \dfrac{(a_{i+1}-b_{i+1})^2+4S_{i+1}}{n(a_i-b_i)^2+4(n-1)S} \geq \dfrac{\sum \limits_{i=1}^n (a_{i}-b_{i})^2+4\sum \limits_{k=1}^nS_{k}}{n(a_1-b_1)^2+4(n-1)S }(1)=\dfrac{\sum \limits_{i=1}^n (a_{i}-b_{i})^2+4S}{n(a_1-b_1)^2+4(n-1)S}$

 Nên ta chỉ cần chứng minh $\dfrac{\sum \limits_{i=1}^n (a_{i}-b_{i})^2+4S}{n(a_1-b_1)^2+4(n-1)S}\geq \dfrac{1}{n-1}\Leftrightarrow \sum_{i=1}^n (a_{i}-b_{i})^2\geq \dfrac{n}{n-1}(a_1-b_1)^2\Leftrightarrow \sum_{i=2}^n (a_{i}-b_{i})^2\geq \dfrac{1}{n-1}\geq (a_1-b_1)^2$

 Áp dụng bất đẳng thức Cauchy-Schwarz ta có $\sum_{i=2}^n (a_{i}-b_{i})^2\geq \dfrac{1}{n-1}\left (\sum_{j=2}^n a_j-\sum_{k=2}^n b_k \right )^2 (2)$

 Mặt khác $\sum_{i=1}^n (a_i-b_i)=0$ nên $\left (\sum_{j=2}^n a_j-\sum_{k=2}^n b_k \right )^2=(a_1-b_1)^2$

 Từ đó ta có điều cần chứng minh ~.~ Mỏi tay

Chỗ khúc $(1)$ thì tử số của mẫu chỉ là $\sum_{k=1}^{n-1}\left ( a_{k}-b_{k} \right )^{2}+\left ( a_{n+1}-b_{n+1} \right )^{2}+\sum_{k=2}^{n}S_{k}+b_{n+1}a_{n+1}$.Đó là lý do mình hỏi có cái quy ước nào cho $a_{n+1}$ và $b_{n+1}$ hay không  :mellow:

 

Còn khúc $(2)$ thì nên viết rõ là $\sum_{i=2}^{n}\left ( a_{i}-b_{i} \right )^{2}\geqslant \frac{1}{n-1}\left ( \sum_{i=2}^{n}\left | a_{i}-b_{i} \right | \right )\geqslant \frac{1}{n-1}\left ( \sum_{k=2}^{n}a_{k}-\sum_{j=2}^{n}b_{j} \right )$




#624685 Inequalities From 2016 Mathematical Olympiads

Đã gửi bởi dark templar on 03-04-2016 - 22:34 trong Bất đẳng thức - Cực trị

Chào các bạn như tiêu đề đã ghi thì trong topic này chúng ta sẽ cùng thảo luận về các bài toán bất đẳng thức đến từ các cuộc thi học sinh giỏi Toán khắp nơi trên thế giới trong năm 2016 và kết thúc topic này (có thể là sau IMO 2016) chúng ta sẽ có một file pdf “Inequalities From 2016 Mathematical” made in VMF để các thành viên có tài liệu tham khảo, hi vọng các bạn ủng hộ. :)

 

Bài 2 (Korea Winter Program Practice Test). Với số nguyên dương $n \geqslant 2$ và $a_i,\,b_i\;(1 \leqslant i \leqslant n)$ là các số thực dương thỏa mãn $\sum_{i=1}^n a_i = \sum_{i=1}^n b_i.$ Chứng minh rằng 

\[\sum_{i=1}^n \frac{(a_{i+1}+b_{i+1})^2}{n(a_i-b_i)^2+4(n-1)\displaystyle \sum_{j=1}^n a_jb_j} \geqslant \frac{1}{n-1}.\]

 

 

Bạn Huyện cho mình hỏi là bài này có ký hiệu $a_{n+1}=a_1$ và $b_{n+1}=b_1$ không vậy ?




#414567 Chuyên đề số học của diễn đàn VMF

Đã gửi bởi dark templar on 24-04-2013 - 11:20 trong Tài nguyên Olympic toán

Mọi người làm ơn cho hỏi sao có part 2 và 4 không tải được nhỉ?

Có phải bạn down các phần dùng để đóng thành sách không ? Mình đã check và vẫn down được bình thường mà bạn ? ;)




#310334 Topic : Bất đẳng thức chứa biến ở mũ

Đã gửi bởi dark templar on 14-04-2012 - 19:57 trong Bất đẳng thức - Cực trị

Anh xin góp cho topic một bài.

Bài 9. Cho các số $x,y,z$ thỏa mãn $0 \leqslant x,y,z \leqslant 2;x + y + z = 3$. Tìm giá trị lớn nhất của:
$$Q = {\left( {1 + {x^2}} \right)^x}{\left( {1 + {y^2}} \right)^y}{\left( {1 + {z^2}} \right)^z}$$

Anh Thành có thể cho em xin lời giải bài này được không ạ ? Bài này em chỉ mới giải xong cho 1 trường hợp $0 \le x \le y \le 1 \le z \le 2$,còn lại trường hợp $0 \le x \le 1 \le y \le z \le 2$ thì......



#356147 Topic : Bất đẳng thức chứa biến ở mũ

Đã gửi bởi dark templar on 23-09-2012 - 14:52 trong Bất đẳng thức - Cực trị

Bài 56: Cho $x_{i} \ge \frac{1}{2}(i=1;2;...)$.Chứng minh:
$$\prod_{i=1}^{n}\left(1+\frac{2x_{i}}{3} \right)^{2x_{i}} \ge \left(\frac{4}{3} \right)^{n}\sqrt{\prod_{i=1}^{n}(x_{i}+x_{i+1})}$$



#310380 Phương trình-hệ phương trình qua các kỳ TS Đại Học

Đã gửi bởi dark templar on 14-04-2012 - 20:52 trong Phương trình - hệ phương trình - bất phương trình

Bài 33: Tìm $a$ để với mọi $b$ hệ sau có nghiệm:
$$\left\{\begin{matrix} (a-1)x^5+y^5=1 & \\ e^{bx}+(a+1)by^4=a^2 \end{matrix}\right.$$

Đề thi thử ĐH của ĐHKHTN Hà Nội năm 2009.
P/s:Mong các bạn tham gia topic đánh số thứ tự bài.



#404259 PT-HPT-BPT Tuyển tập các bài toán sưu tầm từ Mathslink.ro

Đã gửi bởi dark templar on 11-03-2013 - 21:35 trong Phương trình - Hệ phương trình - Bất phương trình

Hình như topic đang đi lạc chủ đề ? Đang ML $\to$ $30-4$ ?

Mong mọi người tập trung hơn vào chủ đề của topic.Chúng ta đang cần tìm 1 lời giải khác ngoài cách sử dụng hàm số. :)



#405347 PT-HPT-BPT Tuyển tập các bài toán sưu tầm từ Mathslink.ro

Đã gửi bởi dark templar on 15-03-2013 - 21:17 trong Phương trình - Hệ phương trình - Bất phương trình

Không biết anh có đáp án 2 bài này chưa ? Bài 3 nghiệm ảo tung ; Bài 2 thì vô nghiệm :(

Tất cả các bài anh post trong topic này đều đã có lời giải nên yên tâm đi nhé . :)
P.s: Nghiệm phức vẫn tính mà em ;)



#414064 PT-HPT-BPT Tuyển tập các bài toán sưu tầm từ Mathslink.ro

Đã gửi bởi dark templar on 21-04-2013 - 09:51 trong Phương trình - Hệ phương trình - Bất phương trình

Bài 12: Cộng theo vế 3 pt đầu bài ta được $2a+b=24$ với $\left\{\begin{matrix} a=x^{2}+y^{2}+z^{2}\\ b=xy+yz+zx \end{matrix}\right.$

Mặt khác trừ theo vế lần lượt các pt cho nhau ta được $\left\{\begin{matrix} (z-y)(x+y+z)=2\\ (y-x)(x+y+z)=2\\ (z-x)(x+y+z)=4 \end{matrix}\right.$

Bình phương các pt của hệ rồi cộng theo vế ta được $2(x^{2}+y^{2}+z^{2}-xy-yz-zx)(x^{2}+y^{2}+z^{2}+2xy+2yz+2zx)=24\Rightarrow (a-b)(a+2b)=12$

Ta có hệ $\left\{\begin{matrix} b=24-2a\\ (a-24+2a)(a+48-4a)=12 \end{matrix}\right.\Leftrightarrow \left\{\begin{matrix} b=24-2a\\ (3a-24)(3a-48)+12=0 \end{matrix}\right.\Leftrightarrow \left\{\begin{matrix} 9a^{2}-216a+1164=0\\ b=24-2a \end{matrix}\right.$

Xem đến đây số nó lẻ quá.  :ohmy: sợ sai :( nên không tính tiếp 

Bài này ra số lẻ mà em ;) Mà tính được $a,b$ rồi em định làm gì tiếp ? Trình bày đầy đủ lời giải nhé. :)

 

 

${\cos ^2}x + {\cos ^2}2x + {\cos ^2}3x + {\cos ^2}4x = \frac{3}{2}$.

 

 

<=> $2\cos^2{x}+2\cos^2{2x}+2\cos^2{3x}+2\cos^2{4x}=3$

 

<=> $1+cos^2{2x}+1+cos^2{4x}+1+cos^2{6x}+1+cos^2{8x}=3$

 

<=> $cos^2{2x}+cos^2{4x}+cos^2{6x}+cos^2{8x}=-1$

Em sử dụng công thức hạ bậc rồi sao còn có số mũ 2 ? Phải là $\cos 2x+\cos 4x+\cos 6x+\cos 8x=-1$.

 

Bài toán 12: Cho hệ PT $\left\{ \begin{array}{l}{x^2} + xy + {y^2} = 6\\{x^2} + xz + {z^2} = 8\\{y^2} + yz + {z^2} = 10\end{array} \right.$.Tìm tất cả các giá trị thực của $y$.

 
Bài toán 13: Tìm nghiệm dương của PT:
$$\sqrt {2 + \sqrt {2 + \sqrt {2 + x} } }  + \sqrt 3 \sqrt {2 - \sqrt {2 + \sqrt {2 + x} } }  = 2x$$

Lời giải bài toán 12: 

Trừ PT đầu với PT thứ 2 ta được $x = \frac{2}{{z - y}} - z - y$.

 

Thế vào PT đầu tiên,ta có:

$ - 4\frac{{z + y}}{{z - y}} + \frac{4}{{{{(z - y)}^2}}} + \frac{{2y}}{{z - y}} = 6 - ({y^2} + yz + {z^2}) =  - 4$

 

Quy đồng ta được $3yz = 3{y^2} + 2 \Rightarrow y \ne 0$ và $z = y + \frac{2}{{3y}}$

 

Thế vào PT thứ 3 ta được $27{y^4} - 72{y^2} + 4 = 0$,từ đó tìm được các giá trị của $y$ là $\boxed{\displaystyle y =  \pm \frac{{\sqrt {12 \pm 2\sqrt {33} } }}{3}}$

 

Lời giải bài toán 13:

ĐKXĐ: $2 \ge \sqrt {2 + \sqrt {2 + x} }  \Leftrightarrow x \le 2$,từ đó suy ra $0 < x \le 2$.

 

Vậy tồn tại $y \in \left[ {0,\frac{\pi }{2}} \right)$ sao cho $x=2\cos y$ và ta lần lượt có:

$\sqrt {2 + x}  = \sqrt {2 + 2\cos (y)}  = 2\cos \frac{y}{2}$

 

$\begin{array}{l}\sqrt {2 + \sqrt {2 + x} }  = 2\cos \frac{y}{4}\\\sqrt {2 + \sqrt {2 + \sqrt {2 + x} } }  = 2\cos \frac{y}{8}\\\sqrt {2 - \sqrt {2 + \sqrt {2 + x} } }  = 2\sin \frac{y}{8}\end{array}$
 
Vậy PT đã cho trở thành:
$\begin{array}{l}2\cos \frac{y}{8} + 2\sqrt 3 \sin \frac{y}{8} = 4\cos y\\\Leftrightarrow \sin \frac{\pi }{6}cos\frac{y}{8} + \cos \frac{\pi }{6}\sin \frac{y}{8} = \cos y\\\Leftrightarrow \sin \left( {\frac{y}{8} + \frac{\pi }{6}} \right) = \cos (y)\\\Leftrightarrow \frac{y}{8} + \frac{\pi }{6} = \frac{\pi }{2} - y\end{array}$
(vì $\frac{y}{8} + \frac{\pi }{6} \in \left[ {0,\frac{\pi }{2}} \right)$)
 
Từ đó suy ra $y = \frac{{8\pi }}{{27}}$ và $\boxed{\displaystyle x = 2\cos \frac{{8\pi }}{{27}}}$ là nghiệm duy nhất.
 
**********
Đề mới:
 
Bài toán 14: Giải PT $x = \sqrt {2 + \sqrt {2 - \sqrt {2 + x} } } $.
 
Bài toán 15: Giải hệ PT $\left\{ \begin{array}{l}{x^3} - 3x - 2 = 2 - y\\{y^3} - 3y - 2 = 4 - 2z\\{z^3} - 3z - 2 = 6 - 3x\end{array} \right.$



#425828 PT-HPT-BPT Tuyển tập các bài toán sưu tầm từ Mathslink.ro

Đã gửi bởi dark templar on 10-06-2013 - 21:08 trong Phương trình - Hệ phương trình - Bất phương trình

Bài toán 41: Với các số thực $x,y,z$ thỏa mãn $x^2-kyz=y^2-kzx=z^2-kxy$.Tìm $k$ và bộ nghiệm $(x;y;z)$ tương ứng.

Lời giải bài toán 41: 

Nếu không có số nào trong 3 số $x,y,z$ bằng nhau thì $ x^2-y^2 =-kz(x-y) $,suy ra $ x+y =-kz\,. $.Tương tự,ta có $y+z=-kx;z+x=-ky$.

 

Cộng tất cả lại,ta được $(k+2)(x+y+z)=0$.Do đó $k=-2$ hay $x+y+z=0$.Nếu $k=-2$,thì $x+y=2z;y+z=2x;z+x=2y$ dẫn tới $x=y=z$,mâu thuẫn.Vậy $x+y+z=0$,suy ra $k=1$.Trong trường hợp này $k=1$ và $x+y+z=0$.

 

Bây giờ,giả sử $y=z$,nghĩa là $ x^2-ky^2 = y^2-kxy\,. $,hay $ (y-x)((k+1)y+x) = 0\,. $.Do đó $x=y=z$ hoặc $y=z$ và $x=-(k+1)y$.

 

Kết luận:

$(\star):k=1$ và $(x;y;z)=(a;b;-a-b)$.

$(\star):k \in \mathbb{R}$ và $(x;y;z)=(a;a;a)$.

$(\star):k \in \mathbb{R}$ và $(x;y;z)$ là hoán vị của $(a;a;-(k+1)a)$.

 

Trong đó $a,b$ là tham số bất kỳ.

 

Bài toán 42: Cho $a,b>1$ thỏa $ab=a+b$.Giải PT sau trên tập $\mathbb{R}$:

$$ x[(b-1)a^x+(a-1)b^x-x-ab+2]=(a^x-1)(b^x-1) $$

Lời giải bài toán 42:

Do $a+b=ab$ và:

$ (b-1)a^x+(a-1)b^x-x-ab+2 $

$ =(b-1)a^x+(a-1)b^x-x+(1-a)+(1-b) $

$ =(b-1)(a^x-1)+(a-1)(b^x-1)-x, $

 

Suy ra PT ban đầu tương đương với:

$ x^2-x((b-1)(a^x-1)+(a-1)(b^x-1))+(a^x-1)(b^x-1)=0. $

 

Do $(a-1)(b-1)=1$ nên ta có:

$ \Delta=((b-1)(a^x-1)+(a-1)(b^x-1))^2-4(a^x-1)(b^x-1) $

$ =((b-1)(a^x-1)-(a-1)(b^x-1))^2, $

 

Hay $x=(b-1)(a^{x}-1)$ hoặc $x=(a-1)(b^{x}-1)$.

 

Ở trường hợp đầu,ta có $ (b-1)a^x-x-(b-1)=0. $ PT này có tối đa 2 nghiệm do $ ((b-1)a^x-x-(b-1))''>0 $,suy ra $x \in \{0;1 \}$ là nghiệm.

 

Từ đó PT có 2 nghiệm là $x \in \{0;1 \}$.

 

====================

Đề mới:

 

Bài toán 43: Giải PT $ x^2.2^{x+1}+2^{|x+3|+2}=x^2.2^{|x-3|+4}+2^{x-1} $

 

Bài toán 44: PT $ {|x^2-1|+x^2+kx=0} $ có duy nhất 2 nghiệm thuộc $(0;2)$,tìm điều kiện của $k$ và chứng minh $\frac{1}{x_1}+\frac{1}{x_2}<4$,với $x_1;x_2$ là 2 nghiệm của PT ban đầu.

-22-